LSAT and Law School Admissions Forum

Get expert LSAT preparation and law school admissions advice from PowerScore Test Preparation.

User avatar
 Dave Killoran
PowerScore Staff
  • PowerScore Staff
  • Posts: 5853
  • Joined: Mar 25, 2011
|
#22753
Complete Question Explanation

Must Be True, SN. The correct answer choice is (E)

This is a difficult problem that requires you to manipulate statements and connect common terms. It's time-consuming.

The first sentence of the stimulus uses the term "unless" and can be diagrammed as follows:

  • AGN = attract greater number of applicants ..... PS = problem solved


    Statement: AGN :arrow: PS

    Contrapositive: PS :arrow: AGN
Remember, the clause modified by "unless" becomes the necessary condition and then the remainder of the statement is negated to form the sufficient condition.

The second sentence requires some manipulation on the part of the test taker. The first part of the sentence, which states that "if the pool of able applicants to nursing school does not increase beyond the current level," can be equated with the phrase "cannot attract a greater number of able applicants than they currently do" in the first sentence:

  • AGN = attract greater number of applicants ..... PNI = pool of applicants not increase


    AGN = PNI

Using the contrapositive of the first sentence results in:

  • PS :arrow: AGN = PNI



Adding this diagram to the remainder of the second sentence we arrive at the following diagram:

  • AGN = attract greater number of applicants ..... PNI = pool of applicants not increase

    LS = lower entrance standards ..... S = shortage of nurses




    ..... ..... ..... ..... ..... LS
    PS :arrow: AGN = PNI :arrow:       or
    ..... ..... ..... ..... ..... S


The final sentence makes it clear that with either a lowering of standards (LS) or a shortage of nurses (S), the high quality of health care cannot be maintained (HQM):
  • ..... ..... ..... ..... ..... LS
    PS :arrow: AGN = PNI :arrow:       or ..... :arrow: ..... HQM
    ..... ..... ..... ..... ..... S



Thus, if problems are not solved (PS), then the high quality of health care cannot be maintained (HQM), and it follows that answer choice (E) is correct. Please note that answer choice (E) utilizes the classic LSAT trick of introducing the sufficient condition in the middle of the answer choice via the use of the word "if." Many students expect to encounter conditional statements in the "usual" order (sufficient condition first, necessary condition second) and fall prey to mis-diagramming answer choices such as (E). For the record, there is no proper or standard order for introducing conditions. Necessary conditions can be introduced first or sufficient conditions can be introduced first, and you can expect to encounter both presentations on the LSAT.

Additionally, because of the complexity of linking the various statements together, in this problem you should expect that the correct answer will link the first and last terms together.



Answer choice (A): This is a Mistaken Negation of the first part of the chain.

Answer choice (B): If the pool does not increase, then there will be lower standards OR a shortage. Since this answer overlooks the shortage, it is incorrect.

Answer choice (C): This is a Mistaken Negation of the chain.

Answer choice (D): If they fail to solve problems, then there will be lower standards OR a shortage. Since this answer overlooks the lower standards, it is incorrect.

Answer choice (E): This is the correct answer choice, for the reasons explained above.
 josuecarolina
  • Posts: 24
  • Joined: Jul 20, 2012
|
#4624
Okay question 12 of lesson 2 homework 2 logical reasoning questions>

#12 This question overwhelmed me. I would definitely have guessed and moved on on a real test.



thanks for any help/input
 Steve Stein
PowerScore Staff
  • PowerScore Staff
  • Posts: 1153
  • Joined: Apr 11, 2011
|
#4626
Hi Josue,

That's a good question, and one that requires a good bit of conditional linking:

The first sentence provides the following:
increase applicants --> solve problems (of low pay, high stress)

And the contrapositive:
NOT solve these problems --> NOT increase applicants

And the second sentence provides the following:
NOT increase applicants --> lower std or fewer nurses

These statements link up as follows:
NOT solve these problems --> NOT increase applicants --> lower std or fewer nurses

And the final sentence provides that either of those last two outcomes (lower standards or fewer nurses) will mean that the current quality cannot be maintained:

..... ..... ..... ..... ..... ..... lower standard --> NOT maintain quality
NOT solve --> NOT increase --> ..... or
..... ..... ..... ..... ..... ..... fewer nurses --> NOT maintain quality

This leads us to correct answer choice E, which provides that if the problems are not solved, the current standard will not be maintained.

Tough one! But with practice you will find yourself much more comfortable with these.

I hope that's helpful--let me know.

Thanks!

~Steve
 josuecarolina
  • Posts: 24
  • Joined: Jul 20, 2012
|
#4643
So steve, it just looks like this one was 'extra' hard. Lot's of steps, and the 'split' in the 2nd to last part is not something I think we have learned yet. I think I will shelve this one for now. Thanks!
 ellenb
  • Posts: 260
  • Joined: Oct 22, 2012
|
#6533
Dear Powerscore,

I just want to know why answer B and D is wrong for this question.

Regards,

Ellen
 Nikki Siclunov
PowerScore Staff
  • PowerScore Staff
  • Posts: 1362
  • Joined: Aug 02, 2011
|
#6544
Neither (B) nor (D) need to be true, because you are presented with two options: if the applicant pool doesn't increase, you need to EITHER lower the entrance standards, OR experience a shortage of nurses. Either of these options will lead to a decreased quality of healthcare, but neither is a must.
 ellenb
  • Posts: 260
  • Joined: Oct 22, 2012
|
#8817
Dear Powerscore,

I just had a quick question in regards to this question. How do I diagram answer choices B, and D? Why and how are they wrong?

Thanks

Ellen
 Steve Stein
PowerScore Staff
  • PowerScore Staff
  • Posts: 1153
  • Joined: Apr 11, 2011
|
#8825
Hi Ellen,

That one presents a series of conditional statements about the state of nursing:

Nursing schools cant get more applicants unless the solve wage and stress problems:

More applicants :arrow: solve wage and stress problems

Contrapositive:

NOT solve problems :arrow: NOT more applicants

If they don't get more applicants, that means either lower standards or a shortage:

..... ..... ..... ..... ..... lower standards
NOT more applicants :arrow: ..... or
..... ..... ..... ..... ..... shortage

Linking this with the contrapositive above, we get the following:

..... ..... ..... ..... ..... ..... ..... ..... ..... lower standards
NOT solve problems :arrow: NOT more applicants :arrow: or
..... ..... ..... ..... ..... ..... ..... ..... ..... shortage

Answer choice B:
NOT more applicants :arrow: lower standards

We cannot confirm that schools will lower their standards because of the "or" in the stimulus diagram--there might be a shortage of nurses instead.

Answer choice D:
NOT solve problems :arrow: shortage

We cannot confirm that this would lead to a shortage, for basically the same reason--the "or." The schools might lower their standards instead of having to deal with a shortage.

I hope that's helpful--please let me know whether this is clear--thanks!

~Steve
 ellenb
  • Posts: 260
  • Joined: Oct 22, 2012
|
#8860
Thanks Steve,

What if I had an and statement for example


A-->B--> D and P

and I know that i have A, than can I conclude that I have D for sure? (because sometimes I know that they like to eliminate the other true condition. For instance, the tennis players sitting in the office logical reasoning must be true question page 1-111 quest 3).

or if I bring my example with the nurses, and have


lower standards
NOT solve problems--> NOT more applicants --> and
shortage

and an answer choice said, It the nursing profession fails to solve its problems (not solve problems) than they would have to lower their standards (or do I have to include the other and of the statement too?)

I am more concerned with what happens when it is an and statement, how dose it change things and what can we conclude for sure (since this is a MBT question).

Thanks in advance

Ellen
 BethRibet
PowerScore Staff
  • PowerScore Staff
  • Posts: 200
  • Joined: Oct 17, 2012
|
#8958
Hi Ellen,

With your first diagram, since it says D and P, then yes, we know both are required, such that if A is present, then D must be present as well.

On the second example, I'm not sure what you're asking with "or do I have to include the other and of the statement too?"

However if you were trying to change the diagram Steve provided from an "or" to an "and", you would basically just change that word, you wouldn't need to add additional arrows.

Hope this helps!
Beth

Get the most out of your LSAT Prep Plus subscription.

Analyze and track your performance with our Testing and Analytics Package.